Mathe-Marathon Schule - Seite 5

Neue Frage »

loler90 Auf diesen Beitrag antworten »

Ich versuche mich mal an der Lösung der Aufgabe smile

Zitat:

Lösung Aufgabe 61
Der gesuchte Primzahlendrilling ist: .
Wie man weiß ist eine Primzahl immer eine ungerade Zahl, daraus folgt wie angegeben für einen Primzahlendrilling, dass nur jede 2. Zahl als Primzahl in Frage kommt. Das wird auch von der Aufgabenstellung gefordert. . Aber wenn man sich die Zahlen anguckt, die aufeinander folgen z.B. , wird man feststellen, dass bei so einem gebildeten Drilling IMMER eine Zahl, die durch 3 teilbar ist dabei sein wird. Hier z.B. . Bei der lösung für diese Aufgabe gibt es eine Besonderheit, eine Zahl ist selber die 3, also ist so die oben aufgestellte Regel umgangen und somit der einzige Primzahlendrilling mit der Vorraussetzung .

Weitere Beispiele:




Ich hoffe, dass die Erläuterung so gut ist smile
Quastor Auf diesen Beitrag antworten »

Als Ergänzung:

Zitat:
Eine der Zahlen n, n+2 und n+4 muss durch 3 teilbar sein, denn per definition gilt: Jede 3. Zahl ist durch 3 teilbar. Nun schreiben wir n+4=(n+1)+3. Die Summe von zwei Zahlen, die beide durch 3 teilbar sind, ist auch wieder durch 3 teilbar. Somit haben n, (n+1)+3 und n+2. Somit ist eine der Zahlen durch 3 teilbar


Die nächste Aufgabe kommt gleich.
Quastor Auf diesen Beitrag antworten »

Zitat:
Aufgabe 62
Bereich: Analytische Geometrie

Beweisen Sie:
Bei jedem Quader haben die Quadrate über drei von einer Ecke ausgehenden Kanten zusammen den gleichen Flächeninhalt wie das Quadrat über der Raumdiagonalen.
Tesserakt Auf diesen Beitrag antworten »

Zitat:
Lösung Aufgabe 62
[attach]27273[/attach]

Ein Quader entstehe durch die senkrechte Verschiebung eines Rechtecks mit den Seitenlängen und um Längeneinheiten, sodass der Quader die Kantenlänge hat und von Rechtecken der Seitenlängen sowie und begrenzt wird.
Ein Quadrat über der Kante hat den Flächeninhalt . Folglich gilt für und : .
Für die Diagonale des Rechtecks mit den Seiten und gilt .
Sei die Raumdiagonale des Quaders.
Nach Voraussetzung liegt im rechtwinkligen Dreieck, welches die Katheten und hat.
Folglich .
Das Quadrat über der Raumdiagonalen hat somit den Flächeninhalt .
q.e.d.

Tesserakt Auf diesen Beitrag antworten »

Zitat:
Aufgabe 63 Algebra
Seien mit sowie .
Fernerhin gelte .
Man zeige:
Equester Auf diesen Beitrag antworten »

Ich versuch mich mal, damits hier weitergeht.

Zitat:
Lösung 64



Es gilt , da

Es muss also gelten .
Das gilt nur für (mit obiger Bedingung, dass )


Wäre das in Ordnung? Oder habe ich etwas vergessen zu beachten? Augenzwinkern
 
 
Tesserakt Auf diesen Beitrag antworten »

Genau so habe ich es mir vorgestellt. smile

Du kannst eine Aufgabe stellen.
Equester Auf diesen Beitrag antworten »

Zitat:
Aufgabe 65

Hans und Franz wollen Mutter beim Plätzchen backen helfen. Dabei sind sie die Herren der Waage.
Als Mutter die Jungs um 2kg Mehl bittet, fällt diese jedoch herunter. Die Balkenwaage notdürftig
wieder hergerichtet, hat sie nun zwei unterschiedlich lange Arme. Doch Hans hat eine Idee:
Er legt ein 1-kg-Gewicht auf die eine Schale der Waage und schüttet in die andere so lange Mehl,
bis Gleichgewicht herrscht. Diese Menge an Mehl in einer Schüssel aufbewahrt, legt er das 1-kg-Gewicht
auf die andere Seite und wiederholt die Prozedur. Stolz verkündert er Mutter und Franz, er habe
2kg abgewogen und zum Backen zur Verfügung. Doch Franz ist sich unsicher.
Wer hat recht?



(Bitte nicht nur mit Namen, sondern auch mit Begründung antworten^^)
Tesserakt Auf diesen Beitrag antworten »

Zitat:
Lösung Aufgabe 65
Eine Balkenwaage basiert auf dem Prinzip des Hebelgesetz.
Eine solche Waage mit sinnvollerweise gleichgroßem Last- und Kraftarm ist im Gleichgewicht, wenn gilt

Da die zu betrachtenden Kräfte Gewichtskräfte sind, ist mit Schwerebeschleunigung und Masse in Kilogramm. Somit gilt

Im vorliegenden Fall sind die Balken der Waage so verschoben, dass sie nicht mehr auf beiden Seiten gleich lang sind. Seien und die sich ergebenden Balkenlängen mit und mit . (I)
Auch hier gilt: Die Waage ist im Gleichgewicht, wenn die Gleichung für zwei Massen in Kilogramm erfüllt ist.

Im ersten Schritt wird eine Masse abgemessen, wobei das Gegengewichtsstück beträgt. Es hänge am Balken der Länge .
Ergo erfüllt die Gleichung , woraus folgt. (II)
Im zweiten Schritt wird eine Masse abgemessen, indem man das Gegengewichtsstück in die andere Waagschale legt, sodass es am Balken der Länge hängt.
Folglich gilt:

Offenbar ist , sodass die abgewogene Masse sich zu
berechnet.

Hans behauptet, er habe abgewogen.
Also .

Betrachten wir erneut Gleichung (II).
Es ist . Man setze ein, sodass sich
ergibt. Daraus lässt sich schlussfolgern.
Nach (II) ist dies äquivalent zu .
Dies stellt jedoch einen Widerspruch zur Voraussetzung dar!

Somit ist die Annahme falsch. Hans hat also nicht abgewogen.
Equester Auf diesen Beitrag antworten »

Man hätte zwar weniger physikalisch argumentieren dürfen, aber es trifft natürlich zu Freude .

Man hätte sogar noch genauer werden können, indem man letztlich behauptet, man hat nicht
nur ungleich 2kg, sondern man hat sogar zu viel:



mit
und wieder ab in oberen Ausdruck.






Trotz des Rätsels Lösung habe ich noch eine Bitte.
Du hattest erst gerade die Chance eine Aufagbe zu lösen (und zu stellen), wie auch direkt davor.
Gib bitte anderen Schülern auch eine Chance überhaupt erst mal über den Thread zu stolpern und/oder
gar darüber zu grübeln Augenzwinkern .
Tesserakt Auf diesen Beitrag antworten »

Nun gut.

Es folgt meine letzte Aufgabe, bevor ich mich für's erste aus diesem Thread zurückziehe. ^^

Zitat:
Aufgabe 66
Man bestimme das , welches die folgende Gleichung erfüllt:
Gast11022013 Auf diesen Beitrag antworten »

Zitat:
Lösung Aufgabe 67:



Substituion von



pq-Formel anwenden:



Rücksubstitution:



Mittels Logarithmus erhält man:



Für den anderen Wert erhält man keine Lösung, da der Logarithmus aus negativen Zahlen nicht definiert ist.


Zitat:
Aufgabe 67
Analysis:

Für jedes ist die Funktion gegeben durch



Das Schaubild von , die x-Achse und die Gerade mit schließen im 1. Feld eine Fläche ein.
Zeigen Sie, dass der Inhalt dieser Fläche von t unabhängig ist.
Von welcher Parallelen zur y-Achse wird diese Fläche halbiert?
srolle Auf diesen Beitrag antworten »

Zitat:

Lösung Aufgabe 68







q.e.d

Nun sei mit die parallele Gerade zur y-Achse, die die Fläche halbiert. Also muss gelten:


Die negative Lösung können wir uns wegen (bzw. der Aufgabenstellung, dass die Fläche im 1. Quadranten eingeschlossen wird) sparen.

Somit halbiert die Fläche.


Zitat:

Aufgabe 69
Lineare Algebra:

Lebensmittel setzen sich aus folgenden Inhaltsstoffen zusammen:
Eiweiß, Kohlenhydrate, Fett, Wasser, Ballaststoffe, sonstige Stoffe. Energieliefernde Nährstoffe (Inhaltsstoffe mit Brennwert) sind dabei Eiweiß, Kohlenhydrate und Fett.

Auf Lebensmittelpackungen sind Nährwert-Tabellen aufgedruckt, die den jeweiligen Anteil von Eiweiß, Kohlenhydraten und Fett sowie den Brennwert je 100g angeben. Die folgende Tabelle gibt diese Werte für Spaghetti, Tomatensoße und H-Milch an:

[attach]27395[/attach]

a) Bestimmen den Brennwert von jeweils 1g Eiweiß, Kohlenhydrate bzw. Fett in der Einheit Kilojoule (kJ).

-----

Aus den drei Lebensmitteln Reis, Putenfleisch und Gemüse werden drei Gerichte (Putengeschnetzeltes mit Reis und Gemüse), (Gemüsereis mit Putenstreifen) und (Fleichspieß mit Gemüse) zubereitet.

Tabelle 1 gibt die Zusammensetzung der Lebensmittel an. In Tabelle 2 werden die für jedes Gericht benötigten Mengen in g angegeben.

[attach]27396[/attach]

b) Ernährungsratgeber empfehlen, pro Tag nicht mehr als 70g Fett zu sich zu nehmen. Wie viel dieser empfohlenen Maximalmenge ist in enthalten? Welches Gericht hat den höchsten prozentualen Anteil an Wasser? Wie hoch ist dieser?


edit von sulo: Zeilenumbruch eingefügt um die Überbreite der Threadseite zu verhindern.
kgV Auf diesen Beitrag antworten »

Damit das vom Tisch wäre:
Zitat:
Lösung Aufgabe 69
a)
Wir wissen:



Ein wenig (viel) herumrechnen: (ich schreib den ganzen Rechenweg jetzt nicht hin- zu viel Tipparbeit smile )



b)

In Fleischspießchen sind 32g Fett enthalten







Damit enthalten die Spießchen mit 73% das meiste Wasser


Zitat:
Aufgabe 70
Bereich: Analysis (Extremwertaufgabe)

[attach]27506[/attach]

Wie müssen die Seitenlängen a und b gewählt werden, um aus dem abgebildeten Werkstück ein Rechteck mit möglichst geringem Verschnitt auszuschneiden?



edit: latex-Tags vervollständigt
Gast11022013 Auf diesen Beitrag antworten »

Zitat:
Lösung Aufgabe 70:

Zielfunktion:



Nebenbedingung:

Denkt man sich das Flächenstück in einem Koordinatensystem, so kann man 2 Punkte exakt bestimmen und daraus eine Geradengleichung erstellen, mit der sich die Grundseiten des Rechtecks beschreiben lassen.



Punkt und Steigung einsetzen um b zu bestimmen:



Da die Gesamthöhe des Werkstückes 5000 ist, ist die verbleibende Höhe an der Stelle x



und die verbleibende Länge



In die Zielfunktion einsetzen:



Wir bilden die ersten beiden Ableitungen:



Für ein Maximum gilt nun:



Somit:



Die zweite Ableitung ist sowieso immer kleiner Null.

Damit haben wir Seitenlängen von



Und einen maximalen Flächeninhalt von:



Zitat:
Aufgabe 71:

ist das Schaubild von mit



mit

mit ist ein beliebiger Punkt auf .
Die y-Achse und die 1. Winkelhalbierende sowie deren Parallelen durch bilden ein Parallelogramm. Berechnen Sie dessen Flächeninhalt und den Umfang .
Zeigen Sie, dass es keinen Wert von gibt, für den der Flächeninhalt maximal wird, dass aber der Flächeninhalt für einen Grenzwert hat.
Bestimmen Sie diesen Grenzwert.


Edit: Wieder mit 1000 multipliziert.
10001000Nick1 Auf diesen Beitrag antworten »

Zitat:


Lösung von Aufgabe 71

Flächeninhalt
Gleichung der Winkelhalbierenden: .
Gleichung der Parallelen zur Winkelhalbierenden:
. Da p durch Q verläuft, setzt man und







Also ist .

Umfang
Sei B der Schnittpunkt von mit .
Sei C der Schnittpunkt von mit der y-Achse .
Sei A der Koordinatenursprung.
Dann ist


Also ist .


Zeigen, dass es keinen Wert von gibt, für den der Flächeninhalt maximal wird


hat eine Extremstelle bei

ist ein Minimum. Weil keine weiteren Extremstellen hat, besitzt die Funktion auch kein Maximum.


Grenzwert





Zitat:


Aufgabe 72:

Bereich: Stochastik

Aus einem Skatspiel (32 Karten: 7, 8, 9, 10, B, D, K, A in jeweils 4 Farben(Kreuz, Pik, Herz, Karo)) werden 4 Karten gezogen (ohne Zurücklegen).
a) Wie groß ist die Wahrscheinlichkeit, dass nur Zahlen dabei sind (also nur 7, 8, 9 oder 10)?
b) Wie groß ist die Wahrscheinlichkeit, dass unter den 4 Karten mindestens ein A ist?
c) Wie groß ist die Wahrscheinlichkeit, dass man entweder nur Karo oder nur A und D zieht?

Viel Spaß! smile



Zitat:

EDIT: Ich habe gerade von einem Nutzer erfahren, dass er die Aufgabenstellung falsch verstanden hat oder dass ich die Aufgabenstellung nicht richtig formuliert habe. Also bei c) steht: "nur A und D". Ich meine damit: Es gibt die Möglichkeiten AAAA, DAAA, DDAA, DDDA, DDDD (natürlich auch in anderen Reihenfolgen). Der Nutzer hatte die Möglichkeiten AAAA und DDDD ausgeschlossen.
Also bitte AAAA und DDDD mit in die Lösung einbeziehen.


Edit Equester: Zeilenumbrüche eingebaut.
srolle Auf diesen Beitrag antworten »

Damit Aufgabe 72 mal weg ist:

Zitat:
Lösung: Aufgabe 72








Wobei der erste Summand für ADDD und DAAA steht, der zweite dagegen für AADD.
Hier die für mich richtige Lösung, da die originale Formulierung (für mich) AAAA & DDDD nicht zulässt:



Hier die, die vom Aufgebensteller verlangt war (inkl. den Fällen AAAA & DDDD):





Falls jemand gerade eine Aufgabe parat hätte, kann er sie gerne posten. smile
Falls nicht, werde ich in knapp einer Stunde eine neue Aufgabe einstellen.
srolle Auf diesen Beitrag antworten »

Zitat:
Aufgabe 73:
Stochastik

Der Spieler und Hobby-Mathematiker Chevalier de Méré, der mit seinen Spielproblemen und ihren Lösungen durch Pascal in die Geschichte der Wahrscheinlichkeitsrechnung eingegangen ist, wunderte sich diesem gegenüber einmal, dass er beim Werfen von drei Laplace-Würfeln die Augensumme 11 häufiger beobachtet hatte als die Augensumme 12, obwohl doch 11 durch die Kombinationen 6-4-1, 6-3-2, 5-5-1, 5-4-2, 5-3-3 und 4-4-3 und die Augensumme 12 durch ebenso viele Kombinationen (nämlich welche?) erzeugt würde.

Steckt in seiner Argumentation ein Fehler und war das Ergebnis seiner Beobachtung von vornherein zu erwarten, oder hat er nur "zufällig" ein ungewöhnliches Ergebnis erhalten? Wie groß ist die Wahrscheinlichkeit für die Augensumme 11 und die Augensumme 12 beim Werfen von drei Laplace-Würfeln?


Sollte für jeden machbar sein und kann hoffentlich mal jemand Neuen dazu motivieren, hier teilzunehmen. smile
Tesserakt Auf diesen Beitrag antworten »

Zitat:
Lösung Aufgabe 73
Die Argumentation ist falsch.
Begründung: Man betrachte folgende Zahlentripel mit :

Auch die Permutationen der aufgeführten Tripel erfüllen obige Bedingung, sodass sich insgesamt Zahlentripel ergeben.
Sei die Wahrscheinlichkeit, als Augensumme zu erhalten, dann gilt .

Fernerhin betrachte man folgende Zahlentripel mit :

Auch die Permutationen erfüllen obige Bedingung, sodass sich insgesamt Tripel ergeben.
Sei die Wahrscheinlichkeit, als Augensumme zu erhalten, dann gilt .
Ergo , was sich mit der Beobachtung deckt.

Tesserakt Auf diesen Beitrag antworten »

Bleiben wir mal thematisch ein wenig bei der letzten Aufgabe.

Zitat:
Aufgabe 74 Kombinatorik
Gegeben seien eindeutig voneinander unterscheidbare Objekte (bspw. Kugeln von unterschiedlicher Farbe, wobei keine die Farbe einer anderen hat). Diesen sollen auf Plätze (bspw. Aufreihung) angeordnet werden.
Man zeige, dass es mögliche Anordnungen gibt.
Monoid Auf diesen Beitrag antworten »

Zitat:
Lösung Aufgabe 74:

Das geht mit vollständiger Induktion recht leicht:



Da gibt es offensichtlich nur eine Möglichkeit. Weil ist, gilt der I.A. .


Nach Annahme ist die Anzahl der Permutationamöglichkeiten für n Objekte n!. Wenn man jetzt ein Objekt hinzu nimmt kann es entweder an den Anfang an die 2. Stelle .... an das Ende der Anordnung stellen. Das sind insgesamt n+1 Möglichkeiten. Weil es zu jeder Möglichkeit auch noch die restlichen n Objekte zu permutieren gilt, hat man im Endeffekt Möglichkeiten.
Tesserakt Auf diesen Beitrag antworten »

Völlig richtig. Du kannst die neue Aufgabe stellen.
Monoid Auf diesen Beitrag antworten »

Ok, danke!

Zitat:

Aufgabe 75:


Im Physikunterricht werden die Temperatureinheiten Celsius und Fahrenheit durchgenommen.
"In Amerika misst man die Temperatur in Fahrenheit. Um eine Celsiustemperatur in Fahrenheit umzurechnen, muss man den Celsiuswert mit multiplizieren und zu dem Produkt 32 addieren ", erklärt der Lehrer. "Rechnet bitte die folgenden Werte um!" Nach einiger Zeit meldet sich Anton: " Aber die Umrechnung ist doch ganz einfach", meint er, " man hat eine Temperatur in Fahrenheit, beispielsweise diese" , Anton schreibt eine positive dreistellige Zahl an die Tafel, " dann streicht man nur die erste Ziffer und hängt sie wieder an das Ende der Zahl, und schon hat man die Temperatur in Celsius ."
" Du Scherzbold", antwortet der Lehrer," ich fürchte das ist die einzige Zahl, befindet deine Methode funktioniert."

Welche Zahl schrieb Anton an die Tafel? Gibt es weitere positive dreistellige Zahlen mit dieser Eigenschaft oder hat der Lehrer Recht?
10001000Nick1 Auf diesen Beitrag antworten »

Zitat:
Lösung 75

Um eine Zahl von Fahrenheit in Celsius umzurechnen, muss man so rechnen:

Angenommen, a ist die Hunderterstelle der Fahrenheittemperatur an der Tafel, b ist die Zehnerstelle und c ist die Einerstelle . Dann muss gelten:





Also ist durch 5 teilbar. Dann muss sein.

Man erhält dann:



Daraus sieht man sofort, dass und sein muss.

Die einzige mögliche Zahl, die an der Tafel stehen kann, ist 527. D.h. der Lehrer hat Recht.
Monoid Auf diesen Beitrag antworten »

Vollkommenrichtig! Freude Nächste Aufgabe bitte !
10001000Nick1 Auf diesen Beitrag antworten »

Zitat:

Aufgabe 76

Sabine hat sechs Packungen mit jeweils 6 Dosen Kosmetik gekauft. (in einer Packung sind die Dosen gelb, in einer Packung blau, in einer Packung rot, in einer Packung grün, in einer Packung orange und in einer Packung violett). Eigentlich soll jede Dose 50g wiegen, aber durch einen Produktionsfehler gibt es eine Packung, in der alle Dosen 60g wiegen. Wie oft muss Sabine mindetens wiegen, um herauszufinden, welche Farbe die schwereren Dosen haben?

Sabine hat eine normale Waage, wo die Masse (digital) angezeigt wird. Also keine Balkenwaage, wo auf beide Seiten etwas gelegt wird und man sieht, welche Seite schwerer ist
Quastor Auf diesen Beitrag antworten »

Zitat:

Lösung 76
Sabine muss min. 2 mal wiegen(eig. 3).
Zuerst legt sie 3 Farben aufeinmal auf die Waage, damit hat sie 2 Stapel, einer von denen ist 10g schwerer (1. mal wiegen). Dann legt sie von dem schwereren Stapel 2 Packungen auf die Waage (2. mal wiegen), dann erwischt sie entweder die 2 gleich schweren und sie wüsste nach 2 mal wiegen, welche die 60g Dosen sind oder sie wiegt noch einmal eine der 2 Dosen (3. mal wiegen), entweder erwischt sie dann die 60 oder 50g Dosen. Wenn es die 50g Dose ist, weiß sie das es die andere sein muss.


Edit:
Da ich da oben puren Müll verzapft habe, nochmal die (hoffentlich) richtige Lösung

Zitat:

Sabine muss nur 1 mal wiegen. Sie nimmt von einer Farbe 1 Dose, von der nächsten 2 Dosen usw., bis sie bei der letzten Packung alle Dosen nimmt. Nun sieht sie, dass es entweder 10g, 20g, 30g, 40g, 50g oder 60g zuviel sind, wenn die Dosen alle jeweils nur 50g wiegen würden. Nun weiß sie, wenn es 10g sind, dass die Farbe wovon sie nur eine Dose genommen hat, die 60g Dosen sind usw..
10001000Nick1 Auf diesen Beitrag antworten »

Richtig. Nächste Aufgabe bitte!
Gast11022013 Auf diesen Beitrag antworten »

Zitat:
Das Rätsel des Diophantos:

Knabe zu sein gewährte im Gott ein Sechstel des Lebens; noch ein Zwölftel dazu, und Er kleidete seine Wangen in Flaum. Ein Siebtel noch, und er entzündete ihm das Licht der Ehe; fünf Jahre nach der Heirat schenkte Er ihm einen Sohn. Doch ach! - das spätgeborene kränkliche Kind: die Hälfte der Lebensspanne des Vaters hatte es erreicht, da raffte das kalte Schicksal es hinweg. Vier Jahre lang fand er Trost in dieser Wissenschaft der Zahlen dann beschloß sein Leben auch er.

Berechne das alter des Diophantos.
10001000Nick1 Auf diesen Beitrag antworten »

Ich bin mir nicht ganz sicher, aber ich probier's mal:

Zitat:

Sei das Alter des Diophantos und das Alter seines Sohnes. dann kann man folgende Gleichungen aufstellen:





Die zweite Gleichung nach umgestellt ergibt:

In die erste Gleichung einsetzen und nach x umstellen. Man erhält dann

Also wurde Diophantos 84 Jahre alt.


Richtig???
10001000Nick1 Auf diesen Beitrag antworten »

Zitat:
Aufgabe 78 (Analysis)

Der Graph einer ganzrationalen Funktion fünften Grades ist punktsymmetrisch zum Koordinatenursprung und besitzt im Punkt ein Extremum. Außerdem ist die Gerade Wendenormale durch genau einen Wendepunkt des Graphen von

Bestimme eine Funktionsgleichung von
Gast11022013 Auf diesen Beitrag antworten »

Zitat:

Lösung Aufgabe 78:
Da die Funktion den Grad 5 hat und punktsymmetrisch ist, hat sie folgende Vorschrift:



Sie geht durch den Koordinatenursprung:

f(0)=0

Im Punkt (0,5|0,5) ist ein Extremum, daher

f(0,5)=0,5

und

f '(0,5)=0

Die Wendenormale geht durch genau einen Wendepunkt. Dieser liegt z.B. im Koordinatenursprung, da die Funktion zu diesem symmetrisch ist.
Somit:

f '(0)=-6

,weil die Steigung der negative Kehrwert von 1/6 ist, damit man die Funktion Rechtwinklig auf der Geraden steht.

Daraus folgt das Gleichungssystem



Nun wird c=-6 in die 3te Gleichung eingesetzt und auf die andere Seite gebracht. Dann ist das zu lösende Gleichungssystem:

Monoid Auf diesen Beitrag antworten »

Zitat:

Aufgabe 79:

Man bestimme alle nicht negativen, ganzzahligen Lösungen der Gleichung .

Tesserakt Auf diesen Beitrag antworten »

Zitat:
Lösung Aufgabe 79

mit


sowie sind Lösungen dieser diophantischen Gleichung.
Ich behaupte nun, dass es keine weiteren Lösungen gibt.

Beweis:
Angenommen, es existieren weitere Lösungen der Form mit . Dann gilt
, also im Widerspruch zur Annahme.
Trivialerweise folgt, dass auch keine Lösungen der Form mit existieren.

Angenommen, es existieren weitere Lösungen der Form mit .
löst nicht die Gleichung, also muss gelten.

Also .
Nach unserer Annahme existiert also ein natürliches mit .

Lemma 1: für alle und .
Beweis: (per vollständiger Induktion)
Induktionsanfang: Für ergibt sich , was wahr ist.

Induktionsvoraussetzung: für ein

Induktionsschritt: zu zeigen:
Es ist nämlich nach der Induktionsvoraussetzung.
Aus folgt .
Angenommen, . Dann ist äquivalent zu , was offenbar wahr ist.
Es folgt also
Hieraus folgt das zu beweisende Lemma.

Aus dem Lemma folgt , was ein Widerspruch zu darstellt.

Es existieren also auch keine Lösungen der Form bzw. .


Existieren also weitere Lösungen, so sind diese von der Form mit .

Es gilt also .

Lemma 2: für ein festes natürliches und alle natürlichen .
Beweis: (per vollständiger Induktion)
Induktionsanfang: Für ergibt sich , also , was nach Lemma 1 wahr ist.

Induktionsvoraussetzung: für ein

Induktionsschluss: zu zeigen:
Es ist nach der Induktionsvoraussetzung.
Angenommen, , dann , was wegen offenbar wahr ist.
Also gilt .
Hieraus folgt das zu beweisende Lemma.

Es folgt also, dass nicht die Gleichung löst, wenn und die Aussage erfüllen.
Analog folgt auch, dass alle mit der Gleichung nicht genügen.

Nun mehr bleibt also das Zahlenpaar zu prüfen.

.

Es folgt also, dass keine Lösungen außer und existieren – q.e.d.

Tesserakt Auf diesen Beitrag antworten »

Zitat:
Aufgabe 80
Man zeige, dass jede ungerade Quadratzahl der Nachfolger einer durch teilbaren Zahl ist.
Monoid Auf diesen Beitrag antworten »

Zitat:

Lösung Aufgabe 80:

Zunächst bemerken wir, dass das Quadrat einer geraden Zahl wiederum gerade ist.
Also ist jede ungerade Qudratzahl das Qudrat einer ungeraden Zahl.



Die Aufgabe kann man zu folgendem umformulieren:
Sei eine ungerade Qudratzahl. Zeige:

Wei jede ungerade Zahl die Form hat, ist x von der Form wobei k, y ganze Zahlen sind.

Daraus folgt die Behauptung. q.e.d.



Ich warte auf eine Bestätigung.
Tesserakt Auf diesen Beitrag antworten »

Hab nur schnell drüber geschaut, aber sieht korrekt aus.

Es steht dir frei, eine neue Aufgabe zu stellen.
Monoid Auf diesen Beitrag antworten »

Ok, dann die Aufgabe:

Zitat:

Aufgabe 81

Stelle die Zahl

a)
b)
c)

als Summe von 4 unmittelbar aufeinander folgenden , positiver, ganzer Zahlen dar.

10001000Nick1 Auf diesen Beitrag antworten »

Zitat:

Lösung 81

a) Die gesuchten Zahlen haben das arithmetische Mittel Also

b) geht nicht. Da von den 4 Summanden 2 ungerade und 2 gerade sind, ist auch die Summe gerade. Folglich kann 2011 nicht die Summe sein.

c) geht nicht. das arithmetische Mittel der Summanden müsste sein. Das ist aber nur mit einer ungeraden Anzahl von Summanden möglich. Man sieht leicht, dass zu klein ist, aber zu groß. Also kann es keine Lösung geben.
Monoid Auf diesen Beitrag antworten »

Korrekt. Freude Nächste Aufgabe bitte!
Neue Frage »
Antworten »



Verwandte Themen

Die Beliebtesten »
Die Größten »
Die Neuesten »